Question #df3e8

1 Answer
Sep 6, 2015

#E(n) = (sum_(i=1)^n 1/i) / n#

#E(5) = 0.45dot(6)#

#E(100) ~~ 0.05187#

Explanation:

#E(n) = (sum_(i=1)^n 1/i) / n = (H(n))/n#

where #H(n)# is the #n#th Harmonic number.

As #n -> oo#, #H(n) - ln(n) -> gamma#, where #gamma# is the Euler-Mascheroni constant. #gamma ~~ 0.5772#

So #H(n) ~~ ln(n) + gamma#

Actually a better approximation is #H(n) ~~ ln(n+1) + gamma#

Hence #E(n) ~~ (ln(n+1) + gamma)/n#

Using a spreadsheet, I found:

#E(5) = (sum_(i=1)^5 1/i) / 5 = 0.45dot(6)#

#E(100) = (sum_(i=1)^100 1/i) / 100 ~~ 0.05187#

Note that the approximation #E(n) ~~ (ln(n+1)+gamma)/n# gives us corresponding values: #0.4378# and #0.05192#

Background

The series:

#sum_(i=1)^oo 1/i = 1 + 1/2 + 1/3 + 1/4 +...#

is called the Harmonic Series.

It is well known that this does not converge, but how quickly does it diverge?

The finite sum #H(n) = sum_(i=1)^n 1/i# is called the #n#th Harmonic Number.

#H(n) = sum_(i=1)^n 1/i = int_1^(n+1) 1/floor(x) dx#

Compare this with the integral:

#int_1^(n+1) 1/x dx = ln(n+1)#

The smooth graph of #1/x# touches each of the steps of #1/floor(x)#.

The total area between these two curves between #1# and #oo# is the Euler-Mascheroni constant #gamma ~~ 0.5772156649#

So we can write:

#gamma = int_1^oo (1/floor(x) - 1/x) dx#